LSAT and Law School Admissions Forum

Get expert LSAT preparation and law school admissions advice from PowerScore Test Preparation.

 Administrator
PowerScore Staff
  • PowerScore Staff
  • Posts: 8916
  • Joined: Feb 02, 2011
|
#32011
Complete Question Explanation
(The complete setup for this game can be found here: lsat/viewtopic.php?t=15241)

The correct answer choice is (A)

Similar to question #10, if J is assigned to area 2, first consider what would happen if O is in 2 with it: that would force J and K together (closing area 2), which moves L to area 3 with M, and leaves P alone in area 1. But this violates the second rule, so we cannot have O and J together in area 2 (this is seen in Template 2):
PT79_Game_#2_#12_diagram 1.png
Next, consider the implications of the last rule. Since O is not assigned to area 2, we need to ensure that J is assigned to a different area than K. Therefore, K is not assigned to area 2. Furthermore, K cannot be in area 3 (ever, as we’ve discussed, and certainly not here), as then L would not be with either K or M. Consequently, K must be assigned to area 1, proving answer choice (A):
PT79_Game_#2_#12_diagram 2.png
Answer choice (B) is incorrect, because L can also be assigned to area 3.

Answer choice (C) is incorrect, because O must be assigned to area 3.

Answer choice (D): This answer choice is incorrect, because P can also be assigned to area 3.

Answer choice (E): This answer choice is incorrect, because P can also be assigned to area 2.
You do not have the required permissions to view the files attached to this post.

Get the most out of your LSAT Prep Plus subscription.

Analyze and track your performance with our Testing and Analytics Package.